Difference between revisions of "2023 AMC 10B Problems/Problem 25"

(redirect)
(Tag: New redirect)
 
(6 intermediate revisions by 2 users not shown)
Line 1: Line 1:
==Problem==
+
#redirect[[2023 AMC 12B Problems/Problem 25]]
A regular pentagon with area <math>1+\sqrt5</math> is printed on paper and cut out. All five vertices are folded to the center of the pentagon, creating a smaller pentagon. What is the area of the new pentagon?
 
 
 
 
 
==Solution 1==
 
 
 
<asy>
 
unitsize(5cm);
 
 
 
// Define the vertices of the pentagons
 
pair A, B, C, D, E;
 
pair F, G, H, I, J;
 
 
 
// Calculate the vertices of the larger pentagon
 
A = dir(90);
 
B = dir(90 - 72);
 
C = dir(90 - 2*72);
 
D = dir(90 - 3*72);
 
E = dir(90 - 4*72);
 
 
 
// Draw the larger pentagon
 
draw(A--B--C--D--E--cycle);
 
 
 
pair O = (A+B+C+D+E)/5;
 
pair AA,OO;
 
real gap = 0.02;
 
AA = A+(0,0);
 
OO = O+(0,0);
 
 
 
draw(AA--OO, blue);
 
 
 
pair OOO, OAO;
 
OOO = O+(gap,0);
 
OAO = (O+A)/2 + (gap,0);
 
 
 
draw(OOO--OAO,green);
 
dot(O);
 
dot((O+A)/2);
 
 
 
label("$r_b$", (O+A)*.7, E,blue);
 
label("$a_s$", (O+A)*.2 +(0+0.18,0.05), E,green);
 
label("$r_s$", O+(-0.175,0.2), E,pink);
 
 
 
 
 
real scaleFactor = 1/1.618; // Adjust this value as needed
 
// Rotate the smaller pentagon by 180 degrees
 
F = (1 - scaleFactor) * (0,0) + scaleFactor * dir(90 + 180);
 
G = (1 - scaleFactor) * (0,0) + scaleFactor * dir(90 - 72 + 180);
 
H = (1 - scaleFactor) * (0,0) + scaleFactor * dir(90 - 2*72 + 180);
 
I = (1 - scaleFactor) * (0,0) + scaleFactor * dir(90 - 3*72 + 180);
 
J = (1 - scaleFactor) * (0,0) + scaleFactor * dir(90 - 4*72 + 180);
 
 
 
// Draw the smaller pentagon
 
 
 
draw(F--G--H--I--J--cycle,red);
 
 
 
draw(arc(O,(H+I)*.5*.6,H*.6));
 
label("$36^\circ$",O+(+0.05,0.15),NW);
 
draw(O--H,pink);
 
</asy>
 
 
 
Let <math>r_b</math> and <math>r_s</math> be the circumradius of the big and small pentagon, respectively. Let <math>a_s</math> be the apothem of the smaller pentagon and <math>A_s</math> and <math>A_b</math> be the areas of the smaller and larger pentagon, respectively.
 
 
 
From the diagram:
 
<cmath>\begin{align*}
 
    \cos{36^\circ} &= \dfrac{a_s}{r_s} = \dfrac{\phi}{2} = \dfrac{\sqrt{5}+1}{4}\\
 
    a_s &= \dfrac{r_b}{2}\\
 
    A_s &= (\dfrac{r_s}{r_b})^2A_b\\
 
    &=(\dfrac{a_s}{\cos{36^\circ} r_b})^2 (1+\sqrt{5})\\
 
    &=(\dfrac{r_b}{\dfrac{\phi}{2} r_b})^2 (1+\sqrt{5})\\
 
    &=(\dfrac{1}{2 \dfrac{\phi}{2}})^2 (1+\sqrt{5})\\
 
    &=(\dfrac{2}{\sqrt{5}+1})^2 (1+\sqrt{5})\\
 
    &=\dfrac{4}{\sqrt{5}+1} \\
 
    &=\dfrac{4(\sqrt{5}-1)}{(\sqrt{5}+1)(\sqrt{5}-1)} \\
 
    &=\sqrt{5}-1
 
\end{align*}</cmath>
 
~Technodoggo
 

Latest revision as of 21:19, 15 November 2023